Thứ Năm, 30 tháng 6, 2016

Một bài toán hay liên quan đến phép vị tự quay

Đề bài: Cho tam giác $ABC$ nội tiếp $(O)$ và $D$ bất kỳ, $DC,DB$ cắt $(O)$ tại $M,N$ khác $C,B$. $E,F$ là đối xứng của $B,C$ lần lượt qua $AM,AN$. $K$ là tâm ngoại tiếp tam giác $DEF$. Chứng minh răng $AK\perp MN$.


Lời giải (Nguyễn Đức Bảo):

Ta có $\angle DME=2\angle AMB+\angle BMC \ , \ \angle DNF=360^\circ-2\angle ANC-\angle BNC$

Mặt khác $2\angle AMB+2\angle ANC+\angle BNC+\angle BMC=2(\angle AMB+\angle ANC+\angle BMC)=2.180^\circ=360^\circ \implies \angle DME=\angle DNF.$

Để ý $\triangle DMB\sim \triangle DNC\implies \frac{DM}{BM}=\frac{DN}{CN}\implies \frac{DN}{NF}=\frac{DM}{ME}\implies \triangle DME\sim \triangle DNF$

$\implies \angle EDF=\angle MDN \ , \ \frac{DE}{DF}=\frac{DM}{DN}\implies \triangle EDF\sim \triangle DMN \implies DFE=\angle DNM.$

$S\equiv DN\cap (DEF)$. Dễ thấy $\angle BAE=2\angle BNM=2\angle BSE$ nên $A$ là tâm ngoại tiếp tam giác $BES\implies AK\perp ES$

Mặt khác $ES\parallel MN$ nên $AK\perp MN.\blacksquare$.

Nhận xét: -Ý tưởng của bài là tìm một đường tròn sao cho A là tâm của đường tròn đó, và ta đã có AE=AB nên ta sẽ vẽ (A,AB) cắt (K) tại S rồi chứng minh SE // MN Mà SE //MN thì $\angle DNM= \angle DFE$ nên ta sẽ để ý hai tam giác DMN và DFE đồng dang.
- Chúng ta cần chứng minh tam giác DNF đồng dạng DME để sử dụng phép vị tự quay, ta có thể dùng góc định hướng để chứng minh $(DM,ME)=(MC,MA)+(MA,ME)=(NC,NA)+(MB.MA)=(DN,AN)+(AN,NF)=(DN,NF)$

-$\angle DNM= \angle DFE=\angle DSE$ nên MN song song ES.


Thứ Tư, 29 tháng 6, 2016

Một cách chứng minh đi qua điểm cố định và mở rộng của bài VMO 2016

Bài toán 1: Cho tam giác ABC nội tiếp (O) cố định, dây BC cố định không là đường kính, A là điểm di động trên cung lớn BC. (K) đi qua B, C cắt AC, AB tại E, F. (ACH) cắt (ABG) tại D. Chứng minh rằng AD luôn đi qua điểm cố định khi A di động.

Giải:

Do (K) và (O) cố định nên tam giác AFC có các góc không đổi, các tam giác AFC được tạo thành khi điểm A di động luôn đồng dạng với nhau, Do H đối xứng F qua C nên ta cũng có tam giác AHC có các góc không đổi khi A di chuyển, như vậy góc AHF= góc ADC không đổi

Mặt khác do tứ giác EFBC nội tiếp:
$\frac{AE}{AF}=\frac{BE}{CF}=\frac{EG}{FH} $
Từ tam giác AFH đồng dạng tam giác AEG. Vậy $\angle BDA=\angle AHF= \angle ADB$
Từ đó DA là phân giác BDC. Nếu DA cắt (L) (Với L là đường tròn ngoại tiếp tam giác DBC  cố định do $\angle BDC=2 \angle AHF$ cố định)
Vậy M thuộc trung trực BC và (L) cố định nên M cố định. Vậy ta có điều phải chứng minh.

Nhận xét: Ở đây chúng ta đã dùng nếu điểm A di chuyển sao cho tam giác AHC luôn đồng dạng với nhau thì khi đó góc của tam giác AHC luôn không đổi là hằng số.
Bài toán 2: Tam giác ABC, trực tâm H, trung điểm M của BC. P là một điểm trên HM, đường tròn đường kính AP cắt CA, CB  tại E, F. Chứng minh rằng tiếp tuyến tại E, F của (K) cắt nhau trên trung trực BC. (Khi P trung với M ta có đề thi VMO 2016)

Lời giải: Cho (K) cắt cắt (ABC) tại S. Thì $\angle ASP=90^o$ theo bài toán cơ bản thì A(SXBC)=-1, chiếu lên (K) cho ta tứ giác SFLE điều hòa. Suy ra điểm T giao của hai tiếp tuyến tại E và F thuộc SL.

Theo tính chất tiếp tuyến: $ \frac{TL}{TS} = \frac{FL^2}{FS^2} $

Bây giờ ta sẽ dùng cách chứng minh của bài toán 1, Ta cố định tam giác ABC, xét P di chuyển ta thấy $\angle FSL=\angle FAL, \angle FLS= \angle FAS$ cũng không đổi khi P di chuyển. Chú ý rằng S cố định do N, H cố định. 

Do đó các tam giác FSL có hai góc không đổi nên chúng tự đồng dạng với tỉ số $\frac{FL^2}{FS^2}$ không dổi.

Nên tỉ số $\frac{TL}{TS}$ không dổi, mà S cố định, L di chuyển trên đường cao cố định, do đó T di chuyển trên đường song song AX cố định. Khi P trùng M thì theo bài toán VMO 2016 ta có T thuộc trung trực BC song song AX. Vậy ta có đpcm

Hoặc nếu không dùng kết quả VMO 2016, thì ta sẽ dùng phép vị tự quay tâm G biến tam giác GYZ thành tam giác GEF nên biến tiếp tuyến của hai đường tròn này là M thành T. Nên GZF đồng dạng tam giác GMT suy ra $\angle GMT=\angle GZF= 180^o - \angle GZA= 180^o- \angle GHA= \angle AHP$ Suy ra TM song song $AH \perp BC$. Vậy ta có đpcm

Đa thức hệ số hữu tỉ

Đề bài: Tìm tất cả đa thức hệ số hữu tỷ $P(n)$ thoả $P(n)\mid 2^{n} - 1$ với mọi số tự nhiên $n$.

Lời giải:

Các kết quả cơ bản dùng cho lời giải
i) $n = \pm 1$ là số nguyên duy nhất thoả mãn $\pm n\mid 2^{|n|} - 1$
ii) $P(n) \mid P(P(n) + n)$
Lưu ý là ta chỉ quan tâm các đa thức nhận giá trị nguyên với $n$ tự nhiên nên ii) vẫn đúng.
Theo đó, ta có $P(n) \mid P(P(n) + n) \mid 2^{P(n) + n} - 1$. Mặt khác, $P(n)\mid 2^{n} - 1$
Tóm lại ta thu được $P(n) \mid 2^{P(n)}$. Suy ra $P(n) = 1$ hoặc $P(n) = -1$ có vô hạn nghiệm, đến đây dễ suy ra rằng $P(x) = 1$ với mọi $x$ hay $P(x) = -1$ với mọi $x$

Chứng minh i)

Giả sử p là ước nguyên tố nhỏ nhất của $n$ theo Định lý Fermat nhỏ:
 
$2^{p-1} \equiv 1 (mod p)$

mà $(p-1, n)=1$ Suy ra $ord_p(2)=1$ hoặc bằng $p-1$ trường hợp bằng 1 là đơn giản.

Xét trường hợp $ord_p(2)=p-1$ Khi đó: $n=(p-1)k+r( 0<r<p-1)$

Suy ra: $2^n \equiv 2^r \equiv 1 (mod p)$ vô lí do $ord_p(2)=p-1 >r$

Vậy ta có đpcm.

ii) Ta dùng bổ đề đơn giản sau: $P(a)-P(b) \vdots a-b$


Thứ Ba, 28 tháng 6, 2016

Tiếp tục về tứ giác có hai đường chéo vuông góc

Bài toán: Cho tứ giác ABCD với hai đường chéo AC, BD vuông góc với nhau tại I. Gọi EFGH là tứ giác bàn đạp của I trên AB, BC, CD, DA. Khi đó tâm ngoại tiếp tứ giác EFGH nằm trên đường thẳng nối trung điểm AC và BD.

Lời giải (theo bạn Huỳnh Bách Khoa):


Thứ Hai, 27 tháng 6, 2016

Bài toán quốc tế về đường tròn nội tiếp

Đề bài: (IMO 2008) Cho ABCD là tức giác lồi sao cho BA $\ne$ BC. $(I_1 ), (I_2 )$ I lần lượt là đường tròn nội tiếp ABC, ADC .Giả sử tồn tại một đường tròn (I) tiếp xúc với tia BA,BC và tiếp xúc với AD,CD.Chứng minh rằng :tiếp tuyến chung ngoài của $(I_1 ), (I_2 )$ cắt nhau tại điểm nằm trên (I).

Lời giải:


Gọi giao điểm của $(I_1 ), (I_2 )$ với AC lần lượt là J,L Ta dễ chứng minh được 2 điều sau :
1) AB+AD=BC+CD. (  Dùng các tính chất của tiếp tuyến cắt nhau cho (I), $(I_1 ), (I_2 )$) )
2)AL=JC.  (Suy ra từ 1)

Vẽ các tiếp tuyến của (I), $(I_1 ), (I_2 )$ và song song với AC,các tiếp tuyến này lần lượt tiếp xúc với 3 đường tròn trên tại Z,M,N.
Khi đó, thì ta có:B,M,L,Z (Do JM là đường kính của $(I_1)$, L là tiếp điểm bàng tiếp, và B là tâm của phép vị tự biến I thành $I_1$) và D,N,J,Z thẳng hàng (tương tự) . Lại có:JM và LN song song và là đường kính$(I_1 ), (I_2 )$ ( Chứng minh trùng và lưu ý $I_1, I_2$ và tâm vị tự của chúng thẳng hàng) nên JN và LN cắt nhau tại tâm vị tự ngoài của $(I_1 ), (I_2 )$ . Theo tính chất của tâm vị tự ngoài thì đó là giao điểm hai tiếp tuyến chung ngoài của $(I_1 ), (I_2 )$ nên ta có đpcm.

Một số tính chất về tam giác ABC có AB+BC=3AC.

Tính chất 1 (IMO shortlist 2005): .Cho tam giác ABC có AB+BC=3AC.Đường tròn (I) nội tiếp tam giác tiếp xúc với AB,AC tại D,E.Lấy K,L là điểm đối xứng của D,E qua I.Khi đó ALKC nội tiếp.

Chứng minh


Gọi H là giao điểm của CK và AB,T là giao điểm của AL và BC.

Ta có các tính chất cơ bản sau:

AH=BD=AC và CT=BE=AC.

$\angle KCA=90^o-\frac{\angle A}{2}=\angle AID$ (Do tam giác AHC cân)

Suy ra tứ giác AKIC nội tiếp

Tương tự tứ giác ALIC nội tiếp.

Hay 5 điểm A, L, K, I, C cùng thuộc đường tròn.

Vậy ta có đpcm.

Tính chất 2 phát biểu: Cho tam giác ABC đường tròn nội tiếp (I), đường tròn bàng tiếp góc A, B, C tương ứng tiếp xúc BC, CA, AB tại M, N, P. Ta có AM, BN, CP đồng quy tại một điểm khi AB+BC=3AC

Ta chứng minh một bổ đề:

Cho tam giác ABC nội tiếp (O), có đường kính AE, BF. Khi đó đường thẳng qua D song song BC và đường thẳng qua E song song AB cắt nhau tại một điểm thuộc (O).

Chứng minh: Gọi hai đường thẳng đó cắt nhau tại F. Do DF song song BC nên DF vuông BD, tương tự EF vuông EB. Vậy tứ giác FDBE nội tiếp hay F thuộc (O).

Trở lại bài toán. ta sẽ lấy hình 1, và dễ thấy M, N, P trong tính chất 2 là E, F, D trong hình trên.

Ta có AL song song EF do có hai tam giác ACT và tam giác CEF cùng cân tại C.

Tương tự CK song song DF. Vậy theo bổ đề trên ta có AL, CK cắt nhau tại 1 điểm thuộc (I).

Ngoài ra ta có bài toán sau: (USA MO 2001)Cho ABC có (I) là đường tròn nội tiếp tiếp xúc với BC,CA lần lượt tại $ D_1, E_1, D_2, E_2,$ là các điểm nằm trên BC,CA sao cho :$C D_2 =B D_1$ và $C E_2 =A E_1$ .Gọi P là giao điểm của $A D_2$ và $B E_2$ .Đường tròn (I) cắt $A D_2$ tại  điểm,Q gần A.(Q là giao của $A D_2$ và (I)).Chứng minh :$AQ= D_2 P.$

Lời giải:

Khó có thể chứng minh trực tiếp được nên nếu ta đưa về tỉ số thì sẽ dễ chứng minh hơn. Mà ta lại có:

$\frac{AQ}{AD_2}=\frac{r}{r_a}$ (r là tâm nội tiếp, $r_a $ là tâm bàng tiếp )

Ta sẽ chứng minh : $\frac{D_2P}{AD_2}=\frac{r}{r_a}$

Vì B, P, $E_2$ thẳng hàng.

Theo menelaus cho tam giác $AD_2C$ thì:

$\frac{D_2P}{AP}=\frac{BD_2.CD_1}{BC.E_2A}=\frac{p-a}{a}$
Suy ra:
$\frac{D_2P}{AD_2}=\frac{p-a}{a}=\frac{r}{r_a}$

Vậy ta có đpcm.

Áp dụng bài toán trên thì nếu cho:

 P là giao của AT và CH. Thì :HP=CK và AL=PT.

Do đó ta sẽ có: tam giác AKC bằng APH  và tam giác CTP bằng tam giác CAL

Từ đây có thể suy ra tính chấtt 1 dễ dàng: $\angle ALC=\angle CPT=\angle APH=\angle AKC$.

Ngoài ra: AK và AT là hai đường đẳng giác trong góc A và CL và CP là hai đường đẳng giác trong góc C. Suy ra L, K là hai điểm đẳng giác của tam giác ABC

Và I là trực tâm APC. Kết hợp với tính chất 2 là P thuộc (I) ta suy ra P đối xứng F qua I.


Chủ Nhật, 26 tháng 6, 2016

Câu dãy số số học trong IMO và phương pháp suy luận

Đề bài: (IMO 18th) Cho dãy $(u_n)$ xác định như sau: $u_o=2$, $u_1=\frac{5}{2}$ và

$u_{n+1}=u_n(u_{n-1}^2-2)-u_1$

Chứng minh rằng $[u_n]=2^{\frac{2^n-(-1)^n}{3}}$

Lời giải:

Do đề bài yêu cầu chứng minh $[u_n]=2^{\frac{2^n-(-1)^n}{3}}$

Nên ta sẽ cố gắng biểu diễn $u_n$ dưới dang $2^x+a$

Bắt đầu với $u_1$

$u_1=\frac{5}{2}=2+\frac{1}{2}$

$u_2=(2+\frac{1}{2})(2^2-2)-(2+\frac{1}{2})=2+\frac{1}{2}$

$u_3=(2+\frac{1}{2})[(2+\frac{1}{2})^2-2]-(2+\frac{1}{2})$

Ta có thể đoán trước được $u_3=2^3+a (a<1)$ do thay 3 vào điều kiện đề bài

Nên ta tìm cách lấy số $2^3$ ra khỏi $u_3$, ta được:

$u_3=2^3+\frac{1}{2^3}$

Tương tự $u_4=2^5+\frac{1}{2^5}$

Bây giờ ta sẽ chứng minh: $u_n=2^{a_n}+\frac{1}{2^{a_n}}$

Do bài toán cần chứng minh $[u_n]=2^{\frac{2^n-(-1)^n}{3}}$  ta sẽ chứng minh:

Với $a_n=\frac{2^n-(-1)^n}{3}$ đây là công thức tổng quát của $a_n$ tuyến tính bậc 2 nên ta viết lại $a_{n+1}=a_n+2a_{n-1}$

Dễ thấy mệnh đề đúng với n=1,2,3,4,5.

Với n+1 thì:

$u_{n+1}=(2^{a_n}+2^{-a_{n}})(2^{a_{n-1}}+2^{-a_{n-1}-2}-(2+\frac{1}{2})$

Nhân ra ta được:

$u_{n+1}=2^{a_n+2a_{n-1}}+2^{-a_{n}-2a_{n-1}}+2^{2a_{n-1}-a_n}+2^{a_n-2a_{n-1}}-2-2^{-1}$

Mặt khác  Do: $a_{n+1}=a_n+2a_{n-1} \Rightarrow 2a_{n-1}-a_n=(-1)^n$

Thay vào ta có đpcm.

Thứ Bảy, 25 tháng 6, 2016

Từ một bổ đề cho đến bài thi ELMO 2016

Ta có bổ đề sau:

Bổ đề . $\triangle ABC$, $M$, $N$ thuộc $BC$ thì $(AMN)$ tiếp xúc $(ABC)$ khi và chỉ khi $AM$, $AN$ đẳng giác với góc $A$.

Chứng minh bổ đề: $O$, $I$ là tâm của $(ABC)$ và $(AMN)$, $AH$ là đường cao của $\triangle ABC$ thì đương nhiên $AH$ cũng là đường cao của $\triangle AMN$. $AH$, $AO$ đẳng giác với $\angle (AB,AC)$; $AH$, $AI$ đẳng giác với $\angle (AM,AN)$ nên $(A,O,I)$ thẳng hàng khi và chỉ khi $\angle(AB,AC)$ và $\angle (AM,AN)$ có chung phân giác - tức là $AM$, $AN$ đẳng giác.

Đề bài. Trong $\triangle ABC$ với $AB \neq AC$, cho đường tròn nội tiếp của nó tiếp xúc với $BC, CA$ và $AB$ tại $D, E$ và $F$, theo đúng thứ tự. Phân giác trong của $\angle BAC$ cắt đường $DE$ và $DF$ tại $X$ và $Y$, theo thứ tự. Gọi $S, T$ là các điểm khác nhau trên cạnh $BC$ sao cho $\angle XSY = \angle XTY = 90^{\circ}$. Cuối cùng, gọi $\gamma$ là đường tròn ngoại tiếp $\triangle AST$.
Chứng minh rằng $\gamma$ tiếp xúc với $\odot (ABC)$
Chứng minh rằng $\gamma$ và đường tròn nội tiếp của $\triangle ABC$ tiếp xúc nhau.

Lời giải:




câu a)

Để ý $D(BA,EF)=-1\implies D(ZA,YX)=-1$ mà $\angle YSX=\angle YTX=90^\circ\implies SX,TX$ lần lượt là phân giác $\angle ZTA,\angle ZSA\implies AI$ là phân giác $\angle SAT\implies AS,AT$ đẳng giác trong $A\implies \gamma $tiếp xúc $(O)$.


Câu b) Do $DE$ đi qua tâm nội tiếp tam giác $AST,(I)$ tiếp xúc với $AC,TC$ tại $E,D$ nên theo bổ đề Sayawama thì $(I)$ tiếp xúc $\gamma$

Dùng bước nhảy Vi-et để giải bài toán chia hết

Đề bài: Chứng minh rằng nếu a,b là 2 số nguyên dương thỏa mãn $4ab-1$ là ước của $(a+b-1)(a+b+1) $thì $a=b$


Lời giải:

$4ab-1 \mid (a+b-1)(a+b+1) \iff 4ab-1\mid (a-b)^2+(4ab-1)$

Đến đây ta suy ra: $k_0=\frac{(a-b)^2}{4ab-1} \in Z$

Đến đây hướng giải bằng bước nhảy vi-et xuất hiện, ta tìm cách loại bỏ số hạng ab ở tử.

$k=2k_0+1=\frac{2(a^2+b^2)-1}{4ab-1}$



Với k=1 thì $k_o=0$ kéo theo $a=b$

Tiếp tục phần $k>1$

Phương trình $ \iff 2a^2-4kba+2b^2-k=0$ (1)

Ta giả sử $a+b$ nhỏ nhất và $a > b$ (Vì nếu $a=b$ thì $k=1$)

Và theo Vi-et thì, tồn tại t sao cho:
$\left\{\begin{matrix}t+a=2kb\\ ta=b^2-\frac{k}{2}\end{matrix}\right.$

$ \Rightarrow a+b \le b+t \Rightarrow t \ge a > b$
$ \Rightarrow 2kb=t+a \le 2t \Rightarrow 2kab \le 2at$

Mà do (1) thì $2kab=a^2+b^2-\frac{k}{2}$ và $2at=2b^2-\frac{k}{2}$

Từ đây suy ra $b \ge a$ Vô lí
Vậy ta có đpcm.

Đề thi ELMO

18th ELMO
Ngày 1 (18/06/2016)
Bài 1. Cookie Monster gọi một số nguyên dương $n$ là crunchy nếu tồn tại $2n$ số thực $x_{1}, x_{2}, \cdots x_{2n}$ (tất cả không bằng nhau), sao cho tổng của $n$ số bất kỳ trong chúng bằng với tích của $n$ số còn lại. Bạn hãy giúp Cookie Monster xác định tất cả các số crunchy.
Bài 2. Oscar tập vẽ các hình. Oscar vẽ một tam giác $ABC$ và điểm $D$ sao cho $DB, DC$ là tiếp tuyến tới đường tròn ngoại tiếp tam giác $ABC$. Gọi $B'$ đối xứng của $B$ qua $AC$, $C'$ là đối xứng của $C$ qua $AB$. Nếu gọi $O$ là tâm ngoại tiếp của tam giác $DB'C'$, hãy giúp Oscar chứng minh rằng $OA \perp BC$.
Bài 4. Big Bird có một đa thức hệ số nguyên thỏa mãn $n$ chia hết $P(2^{n})$ với mọi số nguyên dương $n$. Chứng minh rằng đa thức của Big Bird là đa thức không.
  1. Chứng minh rằng $\gamma$ tiếp xúc với $\odot (ABC)$
  1. Chứng minh rằng $\gamma$ và đường tròn nội tiếp của $\triangle ABC$ tiếp xúc nhau.


Bài 3. Trong hệ trục tọa độ $Oxy$, ta gọi một hình chữ nhật là bình thường nếu tất cả các cạnh song song với trục $x$ hoặc trục $y$, và gọi một tập các điểm là đẹp nếu như hai điểm bất kỳ trong chúng không có chung hoành độ lẫn tung độ. Đầu tiên, Bert chọn một tập đẹp $B$ gồm $2016$ điểm trên mặt phẳng. Để 'hại não' Bert, Ernie chọn một tập $E$ gồm $n$ điểm trên mặt phẳng sao cho $B\cup E$ là một tập đẹp với $2016 + n$ điểm. Bert trở lại và một cách kỳ diệu, phát hiện ra rằng không có hình chữ nhật bình thường nào chứa ít nhất hai điểm trong $B$ và không có điểm nào thuộc $E$ nằm bên trong nó. Cho một tập đẹp $B$ mà Bert chọn, định nghĩa $f(B)$ là số nguyên dương nhỏ nhất $n$ sao cho Ernie có thể tìm ra một tập đẹp $E$ với kích cỡ $n$ phần tử thỏa mãn điều kiện đặt ra. Hãy giúp Bert xác định GTNN và GTLN của $f(B)$.
Ngày 2 (19/06/2016)
Bài 5. Elmo đang tô màu. Đầu tiên Elmo chọn ra một tập $S$ gồm $n > 1$ điểm thẳng hàng. Sau đó với mỗi cặp không thứ tự $\{X, Y\}$ trong $S$, Elmo tô màu đường tròn với đường kính $XY$ sao cho mỗi cặp đường tròn mà giao nhau tại 2 điểm phân biệt thì được tô khác màu. Count von Count muốn đếm số màu mà Elmo đã sử dụng. Với mỗi $n$ cho trước, hỏi Elmo cần phải dùng ít nhất bao nhiêu màu để tô?
Bài 6. Elmo đang học hình học Olympiad. Trong $\triangle ABC$ với $AB \neq AC$, cho đường tròn nội tiếp của nó tiếp xúc với $BC, CA$ và $AB$ tại $D, E$ và $F$, theo đúng thứ tự. Phân giác trong của $\angle BAC$ cắt đường $DE$ và $DF$ tại $X$ và $Y$, theo thứ tự. Gọi $S, T$ là các điểm khác nhau trên cạnh $BC$ sao cho $\angle XSY = \angle XTY = 90^{\circ}$. Cuối cùng, gọi $\gamma$ là đường tròn ngoại tiếp $\triangle AST$.

Nguồn: http://diendantoanhoc.net/topic/161011-18th-elmo-elmo-lives-mostly-outside/?p=642070

Chứng minh rằng: $\frac{a}{ac+1}+\frac{b}{ab+1}+\frac{c}{bc+1}\leq \frac{1}{2}(a^{2}+b^{2}+c^{2})$

Đề bài: Cho a,b,c là 3 số thực dương thỏa mãn abc=1. Chứng minh rằng:

$\frac{a}{ac+1}+\frac{b}{ab+1}+\frac{c}{bc+1}\leq \frac{1}{2}(a^{2}+b^{2}+c^{2})$


Lời giải:

Ta có:

$ \frac{a}{ca+1}+\frac{b}{ab+1}+\frac{c}{bc+1}\leq \frac{1}{2} (a^{2}+b^{2}+c^{2})$
$\iff \frac{ab}{1+b}+\frac{bc}{1+c}+\frac{ca}{1+a}\leq \frac{1}{2} \left(a^{2}+b^{2}+c^{2}\right) .$
Dùng cô si ở mẫu ta được $\frac{ab}{1+b}+\frac{bc}{1+c}+\frac{ca}{1+a}\leq \frac{1}{2} \left(a\sqrt{b}+b\sqrt{c}+c\sqrt{a}\right) . $Ta chỉ cần chứng minh \[\color{red}a\sqrt{b}+b\sqrt{c}+c\sqrt{a}\leq a^2+b^2+c^2.\] Ta có bất đẳng thức sau \[\color{blue}a^2+b^2+c^2\geq ab+bc+ca\]và \[\color{blue}a^2+b^2+c^2\geq a+b+c,\]AM-GM một lần nữa $\color{red}AM-GM:$ \[\color{blue}2(a^2+b^2+c^2)\geq (ab+a)+(bc+b)+(ca+c)\geq 2(a\sqrt{b}+b\sqrt{c}+c\sqrt{a})\]$\color{red}\Longrightarrow$ \[\color{blue}a^2+b^2+c^2\geq a\sqrt{b}+b\sqrt{c}+c\sqrt{a},\]Vậy ta có đpcm

Chứng minh rằng $(a^2-ab+b^2)(b^2-bc+c^2)(c^2-ca+a^2) \le 12$ với $a,b,c$ dương thỏa $a+b+c=3$

Việc dự đoán điểm rơi rất quan trọng trong chứng minh bất đẳng thức nó giúp ta định hướng được cách giải.

Ta dự đoán được điểm rơi là $c=0, b=1, a=2$ và các hoán vị

Nên ta đánh giá bất đẳng thức mà đảm bảo dấu bằng như sau:


Lời giải:

Giả sử $c=min(a,b,c)$ thì:

$b^{2}-bc+c^{2}\leq b^{2}$ và $a^{2}-ca+c^{2}\leq a^{2}$

Suy ra:

$P\leq a^{2}b^{2}(a^{2}-ab+b^{2})\leq \frac{4}{9}.(\frac{(a+b)^{2}}{3})^{3}\leq \frac{4}{9}.(\frac{(a+b+c)^{2}}{3})^{3}=12.$

Đẳng thức xảy ra khi : $a=2,b=1,c=0$ và các hoán vị.

Thứ Sáu, 24 tháng 6, 2016

Chứng minh tồn tại vô số số $n$ thỏa mãn: $n^2+2^n$ chia hết cho 1994

Lời giải:


Để $n^2+2^n$ chia hết cho 1994 thì trước hết n phải chẵn.

Ta chỉ cần chứng minh tồn tại số n sao cho $n^2+2^n$ chia hết cho 997.

Mà theo tiêu chuẩn Euler thì ta có:

$\left ( \frac{-1}{997} \right )=(-1)^{\frac{997-1}{2}}=1 (mod 997)$

Nên 997 có một bội dạng $a^2+1$. Do (996,997)=1 nên tồn tại hệ thặng dư thư gọn có dạng {996.1;996.2,...;996.996} mod 7

Suy ra tồn tại t để $(996t)^2+1$ chia hết  997

Mặt khác $2^{996t} \equiv 1 (mod 997)$

Vậy tồn tại vô số số n sao cho $n^2+2^n \vdots 1994$ 

Điều kiện cần và đủ của tứ giác có hai đường chéo vuông góc trong IMO shorlist 2008

Bài: Cho tứ giác lồi ABCD. Chứng minh rằng tồn tại điểm P nằm trong tứ giác thỏa mãn:

$\widehat{PAB}+\widehat{PDC}=\widehat{PBC}+\widehat{PAD}=\widehat{PCD}+\widehat{PBA}=\widehat{PDA}+\widehat{PCB}=90^o$

Khi và chỉ khi hai đường chéo AC và BD vuông góc.

Lời giải:


Giả sử tồn tại P thỏa mãn đề bài. Gọi M là điểm Miquel của tứ giác toàn phần tạo bởi các đường thẳng AB, BC, CD, DA, R là giao BC với DA

Theo điều kiện đề bài:

$180^o=\widehat{PAB}+\widehat{PDC}+\widehat{PCD}+\widehat{PBA}=360-\widehat{BPA}-\widehat{CPD} \Rightarrow \widehat{BPA}=180^o-\widehat{DPC}$

Mặt khác: $\widehat{CRD}=180^o-\widehat{RDC}-\widehat{RCD}=90^o-\widehat{PDC}-\widehat{PCD}=\widehat{CPD}-90^o \Rightarrow \widehat{ARB}=90^o-\widehat{BPA}$

Tương tự ta cũng có: $\widehat{BQC}=90^o-\widehat{BPC}$

Vậy: $\widehat{CPA}=\widehat{BPA}+\widehat{BPC}=180^o-\widehat{BRA}-\widehat{BQC}=180^o-\widehat{BMA}-\widehat{CMB}=180^o-\widehat{CMA}$

Suy ra tứ giác APCM nội tiếp, tương tự DPBM nội tiếp.

$\widehat{BCA}+\widehat{DBC}=\widehat{PCB}-\widehat{PCA}+\widehat{PBC}+\widehat{PBD}=180^o-\widehat{CPB}+\widehat{PMD}-\widehat{DMA}=180^o-\widehat{CPB}-\widehat{AMD}=180^o-\widehat{CPB}-\widehat{DQA}=90^o$

Vậy $AC \perp BD$

Chiều ngược lại chỉ cần chứng minh tồn tại điểm liên hợp đẳng giác với điểm J giao điểm hai đường chéo, trong tứ giác ABCD. Theo lời giải của bạn Huỳnh Bách Khoa


Thứ Năm, 23 tháng 6, 2016

Dùng AM-GM để chứng minh bất đẳng thức

Đề bài:
Cho a,b,c dương thỏa mãn: $ab+bc+ca=3$.
Tìm GTNN của: $K=\sum \frac{(1+a)^2(1+b)^2}{1+c^2}$

Lời giải:


Ta có: $(1+a)^{2}(1+b)^{2}=\left [ (1+ab)+(a+b) \right ]^{2}\geq 4(1+ab)(a+b)$

Do đó: $\frac{(1+a)^{2}(1+b)^{2}}{1+c^{2}}\geq 4a.\frac{1+b^{2}}{1+c^{2}}+4b.\frac{1+a^{2}}{1+c^{2}}$

$K\geq 4a.(\frac{1+b^{2}}{1+c^{2}}+\frac{1+c^{2}}{1+b^{2}})+4b.(\frac{1+a^{2}}{1+c^{2}}+\frac{1+c^{2}}{1+a^{2}})+4c.(\frac{1+a^{2}}{1+b^{2}}+\frac{1+b^{2}}{1+a^{2}})$

$\Rightarrow K\geq 8(a+b+c)\geq 8.\sqrt{3(ab+bc+ca)}=24$

Dấu = xảy ra <=>$ x = y = z = 1$

Phương pháp tuyến tuyến với đề thi Nga 1990

Đề: Cho $x, y>0$ và $x^2+y^3 \ge x^3+y^4$. Chứng minh rằng $x^3+y^3 \ge 2$

Lời giải:

Từ giả thiết $(x^3-x^2)+(y^4-y^3) \ge 0$

Ta sẽ tìm k sao cho $x^3-1 \le k(x^3-x^2), y^3-1 \le k(y^4-y^3)$

Dấu bằng xảy ra khi $x=y=1$ nên số k thích hợp là số sao cho các đường cong:

$f(t)=t^3-1, g(t)=k(t^3-t^2), h(t)=k(t^4-t^3)$ tiếp xúc nhau tại điểm t=1

Do g(t) và h(t) tiếp xúc nhau tại t=1, nên ta tìm k sao cho f(t) và g(t) tiếp xúc với t=1 là đủ.

Điều kiện để hai đồ thị tiếp xúc với nhau là:

$\left\{\begin{matrix}
f(x_o)=g(x_o) & \\
f'(x_o)=g'(x_o) &
\end{matrix}\right.$

Thay $x_o=1$ vào ta tìm được $k=3$

Ta kiểm chứng: $x^3-1 \le 3(x^3-x^2) \Leftrightarrow (2x+1)(x-1)^2\ge0\\y^3-1\le3(y^4-y^3)\Leftrightarrow (3y^2+2y+1)(y-1)^2\ge0$

Cộng các vế ta có đpcm.

Tương tự ta có thể chứng minh bất đẳng thức sau: http://diendantoanhoc.net/topic/160967-max-hxy/?p=641961

Dùng đạo hàm để chứng minh bất đẳng thức

Đề: Cho $x>0$. Chứng minh rằng $\sqrt{\dfrac{x}{2}+\dfrac{1}{2x}}\ge e^{\left(\frac{x-1}{x+1} \right)^2}$


Lời giải:

BĐT$<=>\ln (\sqrt{\frac{x}{2}+\frac{1}{2x}})\geqslant \frac{(x-1)^2}{(x+1)^2}$ $(*)$

Đặt $f(x)=\ln (\sqrt{\frac{x}{2}+\frac{1}{2x}})- \frac{(x-1)^2}{(x+1)^2}$ $(x>0)$

Ta có: $f'(x)=\frac{x^2-1}{2x(x^2+1)}-\frac{4(x-1)}{(x+1)^3}=(x-1)[\frac{x+1}{2x(x^2+1)}-\frac{4}{(x+1)^3}]=\frac{(x-1)^5}{2x(x+1)^3(x^2+1)}$

$f'(x)=0<=>x=1$ mà $f(x)$ đồng biến trên $x\in (0;\infty)$

Suy ra $f(x)\geqslant f(1)=0$ hay $\ln (\sqrt{\frac{x}{2}+\frac{1}{2x}})- \frac{(x-1)^2}{(x+1)^2}\geqslant 0$

Do đó $(*)$ luôn đúng nên ta có đpcm

Dùng phép vị tự quay để giải bài toán hình học

Bài 1: Cho tứ giác ABCD gọi G là giao điểm của AC và BD. Gọi $O_1, O_2, O_3, O_4$ lần lượt là tâm của $ GAB, GBC. GCD, GDA$. Đường thẳng bất kì qua G cắt $(O_2), (O_4)$ tại J và K. Đường thẳng bất kì khác qua G cắt $(O_1), (O_3)$ tại $S$ và $T$. Gọi M là giao của $O_1O_2$ và $O_3O_4$, U, I là trung điểm ST và JK. Chứng minh rằng $MU=MI$.



Lời giải:

Gọi giao điểm thứ hai của $(O_1), (O_3)$ là Q, $(O_2), (O_4)$ là P. E, F là trung điểm AC, BD. Ta sẽ chứng minh G,E, F, P, Q cùng thuộc $w$

Xét phép vị tự quay tâm P biến B thành D, A thành C nên biến BD thành AC. Do E, F là trung điểm AC và BD nên biến E thành F, Suy ra tam giác APE đồng dạng PBF

Suy ra: $\widehat{PEG}=\widehat{PFG}$ nên tứ giác PGEF nội tiếp

Tương tự ta có tứ giác QGEF nội tiếp.

Ta có $O_2O_4$, $O_1O_3$ là trung trực của PG và GQ. nên M là tâm ngoại tiếp của 5 điểm P, Q, G, E, F.

Mặt khác tiếp tục xét phép vị tự quay tâm P biến S thành T và do I là trung điểm ST nên phép vị tự quay này biến I thành trung điểm F của BD nên tam giác IPS đồng dạng tam giác PFB. Suy ra $\widehat{PIG}=\widehat{PFG}$ Suy ra I thuộc đường tròn tâm M. Tương tự U cũng thuộc đường tròn tâm M.

Vậy MI=MU

Nhận xét: Nếu tứ giác ABCD nội tiếp (O) thì ta có OE và OF lần lượt vuông AC và BD, suy ra M là trung điểm PO. Suy ra OG vuông GP ( vì G thuộc đường tròn đường kính PO) Và đây là đề thi Trung Quốc 1992.

Bài 2 (Đề thi chọn đội tuyển Thụy Sĩ): Cho tam giác ABC nội tiếp (O), trực tâm H. D, E trên AB, AC sao cho D, H, E thẳng hàng và tam giác ADE cân tại A. (ADE) cắt (O) tại G. Chứng minh GH vuông GA.

Lời giải:


Gọi BB', CC' là các đường cao của tam giác ABC. R là giao của (AB'C') và (ABC) thì HMR thẳng hàng và HR vuông AR. Ta sẽ chứng minh R thuộc (ADE).

R là tâm phép vị tự quay biến C' thành B, B' thành C nên biến C'B thành B'C, mặt khác ta có:

$\frac{C'D}{BC'}=\frac{EB'}{CB'}$

Vậy biến D thành E.
DC' cắt EB' tại A nên tứ giác RDEA nội tiếp hay R thuộc (ADE)

Bài 3: Cho tam giác ABC nội tiếp (O). Đường tròn nội tiếp (I) tiếp xúc BC, CA, AB tại D, E, F. X là điểm chính giữa cung BC chứa A. (AIX) lần lượt cắt AB, AC tại Y, Z. (DYZ) cắt (I) tại W và D. Gọi M là trung điểm BC, P là tiếp điểm của đường tròn mixtilinear trong góc A. Chứng minh rằng:
a) BY=BM=CZ=CM
b) A, W, P thẳng hàng.

Lời giải:

Gọi N là giao điểm AI và (O).
a)Rõ ràng X, M, N thẳng hàng Phép vị tự quay tâm A. biến:
YB thành IN, nên: $BY=NI\cdot \frac{BX}{NX}=NB\cdot \frac{BX}{NX}=BM$ Tương tự CM=CZ.
b) Từ (1) suy ra Y, M đối xứng với nhau BI, Z, M đối xứng nhau qua CI suy ra I là tâm của (MYZ) và do:
YZ là trục đẳng phương (MYZ) và (YZDW) nên tâm của (YZDW) và I vuông góc YZ
Mặt khác có tâm của (YZDW) và I vuông WD ( WD là trục đẳng phương) nên:
WD song song YZ. Theo định lý Euler ta lại có:
$IP.IX=2rR=ID.XN$Suy ra tam giác IDP đồng dang tam giác XIN, và do XIN đồng dạng XZC (do phép vị tự quay tâm X) và IDP=IWP (Do D và W đối xứng) vậy:
tam giác IWP đồng dạng tam giác XZC.
$ \angle IPW= \angle XCA= \angle XPA$ nên P, W, A thẳng hàng.
Hay W và D đối xứng nhau qua IP
Bài 4: Cho tam giác ABC có (I) là tâm nội tiếp và các tiếp điểm D, E, F trên BC, CA, AB. đường thẳng qua D vuông EF cắt AB tại X. Gọi T là giao điểm của (ADE) và (ABC). Chứng minh rằng 
a) TF vuông TX
b) Tâm đường tròn nội tiếp của tam giác TEF thuộc (I) (thinhrost1)

Lời giải:

AI cắt (ABC) tại G, Gọi AA' là đường kính của (O). Theo kết quả quen thuộc T, I, A' thẳng hàng.

Phép vị tự quay tâm T biến B thành F, C thành E nên

tam giác TBF đồng dạng tam giác TCE nên :

BD/CD=BF/CE=TB/TC như vậy TD là phân giác của $\angle BTC$. Phép vị tự quay tâm T biến EF thành BC biến (TEF) thành (TBC) nên biến I thành G, mà T,D,G thẳng hàng nên gọi Y là giao TI và EF thì phép vị tự này biến Y thành D, do biến I thành G và TDG, TYI thẳng hàng nên YD song song IG hay YD vuông EF như vậy BF cắt DY tại X thì suy ra tứ giác TFXY nội tiếp suy ra: $ \angle XTF= 90^o$

b) Gọi J là tâm nội tiếp của TEF, K là tâm nội tiếp TBC. Thì phép vị tự quay tâm T biến J thành K. Biến TJ, FJ, EJ thành BK, TK, CK, nên $\ange FIE = \angle BJC$ mà $ \angle BJC=90^0+\angle T/2 =90^0+\angle A /2 = \angle FDE$ Vậy IDFE nội tiếp. ta có đpcm

Dùng định lý Miquel để giải bài toán hình học

Bài 1: Cho tam giác ABC nội tiếp (O). Các tiếp tuyến tại B, C cắt nhau tại T. Đường thẳng qua A vuông góc AT cắt BC tại S. $B_1, C_1$ trên ST (T nằm giữa $B_1$ và $C_1$, $B_1 $nằm giữa S và T) sao cho $B_1T=BT=C_1T$. Chứng minh rằng tam giác $ABC$ đồng dạng tam giác $AB_1C_1$.

Lời giải:

Gọi M là trung điểm BC thì T,M,A,S đồng viên.

Ta cũng có $B_1,C_1,B,C$ đồng viên

Gọi K là giao của $BB_1$ và $CC_1$ thì $\widehat{BKC}=180^o-\widehat{KBC}-\widehat{BCK}=180^o-\widehat{KBC}-\widehat{B_1BT}=\widehat{TBC}$

Tương tự ta suy ra BT và CT là tiếp tuyến của $(KBC)$

Suy ra K thuộc (ABC) (Vì tâm của KBC là giao của đường thẳng qua B và C vuông BT và CT).

Vậy A là giao của (KBC) và (SMT). Gọi J là giao của $CB_1$ và $BC_1$ thì theo định lý Brocard $TJ \perp SSK$ tại A', Theo Pascal đảo cho 6 điểm BBKCJ ta suy ra J thuộc (KBC), mà A' lại thuộc (KJC) (Do IJ vuông SK tại A') Suy ra A' là giao của (KBC) và (SMT) vậy $A' \equiv A$

Hoặc cách khác: Do TA đã vuông SA, nên ta phải chứng minh TA vuông AK ( điều này có thể chứng minh bằng biến đổi góc cho S,A,K thẳng hàng).

Từ đó B là điểm Miquel của tam giác KSC nên tứ giác $ASB_1B$ nội tiếp, Suy ra A là điểm Miquel của $BCC_1B_1SK$. Cuối cùng theo phép vị tự quay tâm A góc quay $\varphi$ tỉ số $k$ ta có diều phải cm.

Mở rộng của bài IMO 2005

Đề bài: Cho tứ giác ABCD có DA, BC không song song, P là giao điểm của các đường chéo AC, BD. M, N chạy trên DA, BC sao cho $\dfrac{DM}{DA}=\dfrac{BN}{BC}$. MN theo thứ tự cắt AC, BD tại Q, R. Chứng minh rằng đường tròn (PQR) đi qua một điểm cố định khác P.

Lời giải:

Gọi O là giao điểm của (PAD) và (PBC) thì khi đó:
Xét phép vị tự quay:
$S(O,k, \varphi ):D\rightarrow B\\:   A\rightarrow C\\:\Rightarrow DA\rightarrow BC\\:M\rightarrow N(Do:\frac{DM}{AM}=\frac{BN}{NV})$

Suy ra tam giác OAC đồng dạng tam giác OMN, suy ra $\widehat{MNO}=\widehat{QCO}$ Suy ra tứ giác ONQC nội tiếp. Nên O là điểm Miquel của tứ giác toàn phần BPCQNR. Hay O $\in (PQR)$

Thứ Tư, 22 tháng 6, 2016

Định lý EGZ

Định lý Erdős Pál, Abraham Ginzburg és Abraham Ziv:

Từ 2n-1 số nguyên cho trước, luôn chọn được n số sao cho tổng của chúng chia hết cho n.

Lời giải:

gọi định lý trên là mệnh đề $\mathcal{E}(n)$

$\blacksquare$ ta chứng minh $\mathcal{E}(p)$ đúng với $p\in \mathbb{P}$

Với $p=2$ thì dễ thấy đúng ta xét với $p$ lẻ

gọi các số trong tập là $a_1,a_2,...,a_{2p-1}$

giả sử không tồn tại $p$ số nào chia hết cho $p$

$\Rightarrow \left ( a_{i_1}+a_{i_2}+...+a_{i_p} \right )^{p-1}\equiv 1(mod\ p)$

$\Rightarrow \sum_{1\le i_1<...<i_p\le 2p-1}\left ( a_{i_1}+a_{i_2}+...+a_{i_p} \right )^{p-1}\equiv \begin{pmatrix} 2p-1\\p \end{pmatrix}(mod\ p)$ $(*)$
ta có
$\begin{pmatrix} 2p-1\\p \end{pmatrix}\equiv \begin{pmatrix} 1\\1 \end{pmatrix}\begin{pmatrix} p-1\\ 0 \end{pmatrix}\not \equiv 0(mod\ p)$
ta sẽ chứng minh vế trái $(*)$ chia hết cho $p$,thật vậy ta có
$\sum_{1\le i_1<...<i_p\le 2p-1}\left ( a_{i_1}+a_{i_2}+...+a_{i_p} \right )^{p-1}=\sum_{s_1+s_2+...+s_p=p-1}\frac{(p-1)!}{s_1!s_2!...s_p!}\sum _{1\le i_1<...<i_p\le 2p-1}a_{i_1}^{s_1}a_{i_2}^{s_2}...a_{i_p}^{s_p}$ (thức ra từ chỉ cần $\sum _{1\le i_1<...<i_p\le 2p-1}a_{i_1}^{s_1}a_{i_2}^{s_2}...a_{i_p}^{s_p}$)
trong các số $s_1,s_2,...,s_p$ sẽ có $m \in \left [ 1,p-1 \right ]$ số bằng $0$ nên sẽ có $\begin{pmatrix} 2p-1-(p-m)\\m \end{pmatrix}=\begin{pmatrix} p+m-1\\ m \end{pmatrix}$ cách chọn các số $a_{i_j}$
mà $s_j=0$ do đó số $a_{i_1}^{s_1}a_{i_2}^{s_2}...a_{i_p}^{s_p}$ sẽ được lặp $\begin{pmatrix} p+m-1\\m \end{pmatrix}$ lần ( Có ý nghĩa: sau khi loại bỏ $p-m$ số có số mũ khác 0 thì cần m số (trong $2p-1-(p-m)$ số) có mũ bằng $0$ (vì $a_i^0..a_n^0=1$))

mặt khác

$\begin{pmatrix} p+m-1\\ m \end{pmatrix}\equiv \begin{pmatrix} 1\\0 \end{pmatrix}\begin{pmatrix} m-1\\m \end{pmatrix}\equiv 0(mod\ p)$ (hoặc có thể dùng định nghĩa nhị thức newton để chứng minh.)

$\Rightarrow p\mid VT_{(*)}$

điều này dẫn đến mâu thuẫn

$\blacksquare$ ta chứng minh nếu $\mathcal{E}(u)$ và $\mathcal{E}(v)$ đúng thì $\mathcal{E}(uv)$ đúng

gọi $2uv-1$ số nguyên dương là $a_1,a_2,...,a_{2uv-1}$

vì $2uv-1>2u-1$ nên tồn tại $u$ số có tổng chia hết cho $u$ và gọi tổng đó là $b_1$

lặp lại $2v-2$ lần ta có các tổng $b_1,b_2,...,b_{2v-2}$ chia hết cho $u$

do đó còn lại $2uv-1-u(2v-2)=2u-1$ số thì chọn được tổng $b_{2v-1}$ số chia hết cho $u$

mặt khác trong các số $b_1,b_2,...,b_{2v-1}$ ta sẽ chọn được $v$ số chia hết cho $v$

mặt khác $v$ số này cũng chia hết cho $u$ nên $uv$ số này $($ do mỗi tổng có $u$ số $)$ chia hết cho $uv$

vậy bài toán được chứng minh hoàn toán

Thứ Hai, 20 tháng 6, 2016

Dùng tính chất bậc của đa thức để giải phương trình hàm đa thức.

Bài toán: Tìm tất cả các đa thức P(x) với hệ số thực thỏa mãn:

$(x-1)P(x+1)-(x+1)P(x-1)=4P(x), \forall x\in \mathbb{R}$

Nguồn: http://diendantoanhoc.net/topic/160838-t%C3%ACm-%C4%91a-th%E1%BB%A9c-px-th%E1%BB%8Fa-m%C3%A3n/#entry641463

Lời giải:

Nếu $P(x)$ là đa thức hằng thì dễ thấy $P(x)=0 (\forall x)$ thỏa mãn.

Nếu $P(x)$ là không phải là đa thức hằng, không mất tính tổng quát giả sử P(x) là một đa thức có hệ số cao nhất là 1.


Giả sử $deg P =n$

Khi đó VT: hệ số cao nhất của đa thức là $2(n-1)x^n$ còn VP là $4x^n$ nên $n=3$.

Đặt: $P(x)=x^3+ax^2+bx+c$, bằng đồng nhất hệ số ta được $a=c=0$, $b=-1$

Vì thế tất cả $P(x)$ thỏa mãn là $\boxed{P(x)=ax^3-ax}$ $\forall x$ với a là số thực nào đó.

Nhận xét; Việc giả sử đa thức P(x) là monic (có hệ số cao nhất bằng 1) là được vì do $P(x)$ là đa thức hệ số thực, nên nếu có hệ số cao nhất là a thì chia tất cả các hệ số cho a ta sẽ được đa thức P(x) monic

Chủ Nhật, 19 tháng 6, 2016

4 Cách chứng minh cho bài hình học APMO 2000

Bài toán: Cho \displaystyle \Delta ABC với trung tuyến \displaystyle AM và phân giác \displaystyle AN. Đường thẳng vuông góc với \displaystyle AN tại \displaystyle N cắt \displaystyle AB,AMlần lượt tại \displaystyle P,Q. Đường thẳng vuông góc với \displaystyle AB tại \displaystyle P cắt\displaystyle AN tại \displaystyle O. Chứng minh: \displaystyle OQ\bot BC.


Lời giải 1.
1_Fotor


Một cách tự nhiên ta nghĩ tới việc kéo dài \displaystyle PQ cắt \displaystyle AB tại H.
Ta xét thêm PQ cắt \displaystyle Ax tại \displaystyle K (\displaystyle Ax\parallel BC)
Ta thấy: \displaystyle M là trung điểm của \displaystyle BC mà \displaystyle Ax\parallel BC \displaystyle \Rightarrow (PHQK)=-1 (Tính chất hàng điểm điều hòa)
\displaystyle \Rightarrow \overline{NQ.}\overline{NK}=N{{H}^{2}}=N{{P}^{2}} (Hệ thức Newton)
Mà \displaystyle N{{P}^{2}}=\overline{NA.}\overline{NO}
\displaystyle \Rightarrow \overline{NQ}.\overline{NK}=\overline{NA}.\overline{NO}
Mà \displaystyle KN\bot AO \displaystyle \Rightarrow Q là trực tâm của \displaystyle \vartriangle AOK
\displaystyle \Rightarrow OQ\bot AK\Rightarrow OQ\bot BC (đpcm).
Lời giải 2




Lời giải 3 (Khá giống lời giải 1 nhưng ý tưởng là chứng minh trùng và dùng cực, đối cực)


Giả sử đường thẳng qua O vuông góc BC cắt PR tại Q', Gọi E là giao điểm của đường thẳng qua A song song BC (đặt là l ) và PR.
Xét cực đối cực với (O)
Ta có Q thuộc đường đối cực của A nên A thuộc đường đối cực của Q nên l là đường đối cực của Q. (do l vuông góc OQ).

Từ đó do E là cực của AQ (Do đường đối cực của A là PR, đường đối cực của Q là l)

Gọi M' là giao AQ và BC. thì ta có:

 $(P, R; Q', E) = A(P, R; Q', E) = A(B, C; M', \infty) = -1$

Vậy M' là trung điểm BC. Suy ra Q trùng Q'. đpcm

Cách 4: Giả sử đường thẳng qua C và song song với AN cắt tia BA tại C'. 
Từ B kẻ đường vuông góc C'C cắt C'C tại T và tia AN tại U. 
Gọi A' là trung điểm CC'
Qua C kẻ đường thẳng song song với BT, đường này cắt bA ở V
Gọi chân đường thẳng vuông góc kẻ từ V xuống BT là W. Như thế CVWT là hình chữ nhật
AU là phân giác góc CAV và CV vuông góc với AU, nên U là trung điểm WT. 
Suy ra giao điểm N của AU và CW là tâm của hình chữ nhật CVWT, N chính là trung điểm CW. 
Ta đã có M là 
trung điểm BC. Vì M, N lần lượt là trung điểm các cạnh CB và CW của tam giác CBW nên
MN=BW/2
Do CC' song song AN ta có:
$\angle CC'A= \angle BAN= \angle CAN = \angle C'CA$
do đó AC'=AC.  Khi đó AU=C'T-C'A'. Mà N là tâm của hình chữ nhật CTWV nên $NU=\frac{1}{2}CT$ và

$AN=AU-NU=C'T-C'A'-\frac{1}{2}CT=\frac{1}{2}C'T$

Suy ra: $\frac{MN}{AN}=\frac{BW}{C'T}$ Nhưng MN song song với BW và NP nên:

$\frac{QN}{AN}=\frac{MN}{AN}=\frac{BW}{C'T}$

Bây giờ, ta có AN// VW và NP//BW, do đó các tam giác ANP và tam giác VWB đồng dang, và suy ra:

$\frac{AN}{NP}=\frac{VW}{BW}=\frac{CT}{BW}$

Từ đó ta được: $\frac{QN}{NP}=\frac{QN}{AN}.\frac{AN}{NP}=\frac{CT}{C'T}$

PN là đường cao của tam giác vuông ANO, nên hai tam giác ANP và PNO đồng dang. Từ đây ta suy ra:
$\frac{NP}{NO}=\frac{NA}{NP}$

Mặt khác, hai tam giác ANP và C'TB đồng dạng nên ta có:

$\frac{NA}{NP}=\frac{C'T}{BT}$

Do đó $\frac{QN}{NO}=\frac{QN}{NP}.\frac{NP}{NO}=\frac{CT}{C'T}.\frac{C'T}{BT}=\frac{CT}{BT}$

Ta lại có $\angle CTB$ và $\angle QNO$ là hai góc vuông nên hai tam giác $QNO$ và CTB đồng dạng, mà $NO 'perp BT$, do đó $QO \perp BC$

Vận dụng phương pháp tọa độ kết hợp với phương pháp tổng hợp để giải bài toán hình học

Đôi khi sự kết hợp giữa phương pháp tọa độ và phương phương tổng hợp sẽ giúp cho lời giải ngắn gọn và đẹp hơn. Chúng ta xét ví dụ sau:

Đề bài: (IMO 2000) Cho hai đường tròn $(O_1), (O_2)$ cắt nhau tại M, N. Tiếp tuyến chung gần M của hai đường tròn tiếp xúc $(O_i)$ tại $A_i$, Đường thẳng qua $M$ song song $A_1A_2$ cắt $(O_i) $ở $B_i$, các đường $A_iBi$ cắt nhau tại C, các đường $A_iN$ cắt $B_1B_2$ ở D, E. Chứng minh CD=CE.

Lời giải:


Chọn hệ trục tọa độ $A_1xy$ sao cho $A_1(0;0), A_2(a;0), O_1(0;r_1), O_2(a;r_2)$. Giả sử trong hệ trục $M(s;t)$ Khi đó $B_1(-s;t), B_2(2a-s;t)$ (Tính đối xứng). Từ đó $B_1B_2=2a=2A_1A_2$ Mà $A_1A_2 || B_1B_2$ nên $A_1, A_2$ là trung điểm $B_1C$, $B_2C$, do đó $C(s;-t)$. Vậy $\overrightarrow{CM}=(0;2t),\overrightarrow{B_1B_2}=(2a;0)$, suy ra $CM \perp B_1B_2$, hay $CM \perp DE$

Gọi K là giao MN và $A_1A_2$ ta có:
K là trung điiểm $A_1A_2$ (Dễ cm theo phương tích, trục đẳng phương)
Do $A_1A_2 || B_1B_2$ nên M là trung điểm DE.

Suy ra CM là trung trực của DE. đpcm


Thứ Sáu, 17 tháng 6, 2016

Một cách giải hay cho bài bất đẳng thức Iran 1996

Problem: (Iran 1996) Chứng minh 

Sau đây tôi xin giới thiệu lời giải mà tôi cho là hay nhất, thông minh nhất:

Lời giải ( bởi Võ Quốc Bá Cẩn):
Giả sử . Ta có bổ đề


Chứng minh: Bổ đề tương đương với:


Nhưng vì  nên 
& Vậy bổ đề được chứng minh

Dùng bổ đề trên



.
Dễ thấy theo AM-GM

Bất đẳng thức tuyển sinh lớp 10 chọn lọc

Trong bài viết này, tác giả giới thiệu một số bài BĐT nhẹ nhàng nhưng ý tưởng tương đối mới, mức độ phù hợp với đề thi tuyển sinh vào lớp...